Holomorphic function on a upper half plane that is scale invariant with respect to a positive real number.












1












$begingroup$


Let $theta in mathbb{R}^{+}$ be a fixed number and $theta neq 1$. Let $f:mathbb{H} to mathbb{C}$ be a holomorphic function on the upper half plane such that $forall z in mathbb{H}$, $f(z)=f(theta z)$.

Then, must $f$ be a constant? (I highly doubt it since $f$ is not defined at $0$, but I don't know how to rigorously construct a counterexample.)










share|cite|improve this question











$endgroup$












  • $begingroup$
    Do you mean to say that $f(z) = f(theta z), ; forall theta in Bbb R^+$, so that $f(z)$ constant on each ray $theta z_0$, $z_0 in Bbb H$?
    $endgroup$
    – Robert Lewis
    Jan 7 at 22:33








  • 1




    $begingroup$
    $f(z) = f(cz)$ iff $f(e^z)$ is $log c$ periodic
    $endgroup$
    – reuns
    Jan 8 at 9:02












  • $begingroup$
    @RobertLewis $theta$ is fixed. I am sorry for the confusion.
    $endgroup$
    – chbe
    Jan 10 at 10:09
















1












$begingroup$


Let $theta in mathbb{R}^{+}$ be a fixed number and $theta neq 1$. Let $f:mathbb{H} to mathbb{C}$ be a holomorphic function on the upper half plane such that $forall z in mathbb{H}$, $f(z)=f(theta z)$.

Then, must $f$ be a constant? (I highly doubt it since $f$ is not defined at $0$, but I don't know how to rigorously construct a counterexample.)










share|cite|improve this question











$endgroup$












  • $begingroup$
    Do you mean to say that $f(z) = f(theta z), ; forall theta in Bbb R^+$, so that $f(z)$ constant on each ray $theta z_0$, $z_0 in Bbb H$?
    $endgroup$
    – Robert Lewis
    Jan 7 at 22:33








  • 1




    $begingroup$
    $f(z) = f(cz)$ iff $f(e^z)$ is $log c$ periodic
    $endgroup$
    – reuns
    Jan 8 at 9:02












  • $begingroup$
    @RobertLewis $theta$ is fixed. I am sorry for the confusion.
    $endgroup$
    – chbe
    Jan 10 at 10:09














1












1








1


1



$begingroup$


Let $theta in mathbb{R}^{+}$ be a fixed number and $theta neq 1$. Let $f:mathbb{H} to mathbb{C}$ be a holomorphic function on the upper half plane such that $forall z in mathbb{H}$, $f(z)=f(theta z)$.

Then, must $f$ be a constant? (I highly doubt it since $f$ is not defined at $0$, but I don't know how to rigorously construct a counterexample.)










share|cite|improve this question











$endgroup$




Let $theta in mathbb{R}^{+}$ be a fixed number and $theta neq 1$. Let $f:mathbb{H} to mathbb{C}$ be a holomorphic function on the upper half plane such that $forall z in mathbb{H}$, $f(z)=f(theta z)$.

Then, must $f$ be a constant? (I highly doubt it since $f$ is not defined at $0$, but I don't know how to rigorously construct a counterexample.)







complex-analysis






share|cite|improve this question















share|cite|improve this question













share|cite|improve this question




share|cite|improve this question








edited Jan 10 at 10:02







chbe

















asked Jan 7 at 22:08









chbechbe

83




83












  • $begingroup$
    Do you mean to say that $f(z) = f(theta z), ; forall theta in Bbb R^+$, so that $f(z)$ constant on each ray $theta z_0$, $z_0 in Bbb H$?
    $endgroup$
    – Robert Lewis
    Jan 7 at 22:33








  • 1




    $begingroup$
    $f(z) = f(cz)$ iff $f(e^z)$ is $log c$ periodic
    $endgroup$
    – reuns
    Jan 8 at 9:02












  • $begingroup$
    @RobertLewis $theta$ is fixed. I am sorry for the confusion.
    $endgroup$
    – chbe
    Jan 10 at 10:09


















  • $begingroup$
    Do you mean to say that $f(z) = f(theta z), ; forall theta in Bbb R^+$, so that $f(z)$ constant on each ray $theta z_0$, $z_0 in Bbb H$?
    $endgroup$
    – Robert Lewis
    Jan 7 at 22:33








  • 1




    $begingroup$
    $f(z) = f(cz)$ iff $f(e^z)$ is $log c$ periodic
    $endgroup$
    – reuns
    Jan 8 at 9:02












  • $begingroup$
    @RobertLewis $theta$ is fixed. I am sorry for the confusion.
    $endgroup$
    – chbe
    Jan 10 at 10:09
















$begingroup$
Do you mean to say that $f(z) = f(theta z), ; forall theta in Bbb R^+$, so that $f(z)$ constant on each ray $theta z_0$, $z_0 in Bbb H$?
$endgroup$
– Robert Lewis
Jan 7 at 22:33






$begingroup$
Do you mean to say that $f(z) = f(theta z), ; forall theta in Bbb R^+$, so that $f(z)$ constant on each ray $theta z_0$, $z_0 in Bbb H$?
$endgroup$
– Robert Lewis
Jan 7 at 22:33






1




1




$begingroup$
$f(z) = f(cz)$ iff $f(e^z)$ is $log c$ periodic
$endgroup$
– reuns
Jan 8 at 9:02






$begingroup$
$f(z) = f(cz)$ iff $f(e^z)$ is $log c$ periodic
$endgroup$
– reuns
Jan 8 at 9:02














$begingroup$
@RobertLewis $theta$ is fixed. I am sorry for the confusion.
$endgroup$
– chbe
Jan 10 at 10:09




$begingroup$
@RobertLewis $theta$ is fixed. I am sorry for the confusion.
$endgroup$
– chbe
Jan 10 at 10:09










2 Answers
2






active

oldest

votes


















1












$begingroup$

No, $f$ does not need to be constant.



Define $g$ in the strip $S = { w mid 0 < operatorname{Im} w < i pi } $ as
$g(w) = f(e^w)$. Then for a given positive real number $theta ne 1$
$$
f(z) = f(theta z) quad text{for all } z in Bbb H
$$

if and only if
$$
g(w) = g(w + log theta) quad text{for all } w in S
$$

so that the problem reduces to find all $log theta$ - periodic functions in the strip $S$. The most simple example would be
$$
g(w) = exp left(frac{2 pi i w}{log theta} right)
$$

corresponding to
$$
f(z) = exp left(frac{2 pi i log z}{log theta} right)
$$

where $log z = log |z| + i operatorname{Arg} z$ is the main branch of the logarithm with $0 < operatorname{Arg} z < pi$ for $z in Bbb H$.



All $log theta$ - periodic functions in $S$ are obtained by composing $g$ with a holomorphic function, this leads to the general solution
$$
f(z) = h left(exp left(frac{2 pi i log z}{log theta} right) right)
$$

where $h$ is holomorphic in the annulus ${ w mid exp left(frac{-2 pi^2}{log theta}right) < |w| < 1 }$.






share|cite|improve this answer











$endgroup$













  • $begingroup$
    Thank you so much! It's a great help!
    $endgroup$
    – chbe
    Jan 10 at 10:05



















0












$begingroup$

If we write



$f(z) = f(x + iy) = u(x, y) + iv(x, y) = u(r, phi) + iv(r, phi), tag 1$



where $(r, phi)$ are standard polar coordinates on $Bbb H$, then we have



$dfrac{partial u}{partial r} + i dfrac{partial v}{partial r} = dfrac{partial f(z)}{partial r} = displaystyle lim_{Delta theta to 0} dfrac{f((1 + Delta theta)z) - f(z)}{Delta theta} = lim_{Delta theta to 0} dfrac{f(z) - f(z)}{Delta theta} = 0, tag 2$



since varying $theta$ moves $theta z$ in the radial direction; thus,



$dfrac{partial u}{partial r} = dfrac{partial v}{partial r} = 0; tag 3$



now according to the Cauchy-Riemann equations in the polar coordinates $(r, phi)$,



$ dfrac{partial v}{partial phi} = rdfrac{partial u}{partial r} = 0, tag 4$



$ dfrac{partial u}{partial phi} = -r dfrac{partial v}{partial r} = 0; tag 5$



these two equations together with (3) show that



$nabla u(z) = nabla v(z) = 0, forall z in Bbb H; tag 6$



it follows that $u(z)$ and $v(z)$ are constant in $Bbb H$, since it is a connected open set; thus so also $f(z)$ is constant on $Bbb H$.



The above derivation may be written in a slightly more compact fashion if we recall the polar form of the Cauchy-Riemann equations may be expressed in terms of the single equation in $f$,



$dfrac{partial f}{partial r} = dfrac{1}{ir} dfrac{partial f}{partial phi}; tag 7$



then (2) shows that



$dfrac{partial f}{partial r} = 0, tag 8$



from which we find that



$dfrac{partial f}{partial phi} = 0, tag 9$



and the constancy of $f(z)$ immediately follows.






share|cite|improve this answer









$endgroup$









  • 2




    $begingroup$
    I think that OP means that $f(theta z) = f(z)$ for all $z$ and some fixed $theta in Bbb R^+ - {1}$, not for all $z$ and all $theta$.
    $endgroup$
    – Travis
    Jan 8 at 9:06












Your Answer








StackExchange.ready(function() {
var channelOptions = {
tags: "".split(" "),
id: "69"
};
initTagRenderer("".split(" "), "".split(" "), channelOptions);

StackExchange.using("externalEditor", function() {
// Have to fire editor after snippets, if snippets enabled
if (StackExchange.settings.snippets.snippetsEnabled) {
StackExchange.using("snippets", function() {
createEditor();
});
}
else {
createEditor();
}
});

function createEditor() {
StackExchange.prepareEditor({
heartbeatType: 'answer',
autoActivateHeartbeat: false,
convertImagesToLinks: true,
noModals: true,
showLowRepImageUploadWarning: true,
reputationToPostImages: 10,
bindNavPrevention: true,
postfix: "",
imageUploader: {
brandingHtml: "Powered by u003ca class="icon-imgur-white" href="https://imgur.com/"u003eu003c/au003e",
contentPolicyHtml: "User contributions licensed under u003ca href="https://creativecommons.org/licenses/by-sa/3.0/"u003ecc by-sa 3.0 with attribution requiredu003c/au003e u003ca href="https://stackoverflow.com/legal/content-policy"u003e(content policy)u003c/au003e",
allowUrls: true
},
noCode: true, onDemand: true,
discardSelector: ".discard-answer"
,immediatelyShowMarkdownHelp:true
});


}
});














draft saved

draft discarded


















StackExchange.ready(
function () {
StackExchange.openid.initPostLogin('.new-post-login', 'https%3a%2f%2fmath.stackexchange.com%2fquestions%2f3065559%2fholomorphic-function-on-a-upper-half-plane-that-is-scale-invariant-with-respect%23new-answer', 'question_page');
}
);

Post as a guest















Required, but never shown

























2 Answers
2






active

oldest

votes








2 Answers
2






active

oldest

votes









active

oldest

votes






active

oldest

votes









1












$begingroup$

No, $f$ does not need to be constant.



Define $g$ in the strip $S = { w mid 0 < operatorname{Im} w < i pi } $ as
$g(w) = f(e^w)$. Then for a given positive real number $theta ne 1$
$$
f(z) = f(theta z) quad text{for all } z in Bbb H
$$

if and only if
$$
g(w) = g(w + log theta) quad text{for all } w in S
$$

so that the problem reduces to find all $log theta$ - periodic functions in the strip $S$. The most simple example would be
$$
g(w) = exp left(frac{2 pi i w}{log theta} right)
$$

corresponding to
$$
f(z) = exp left(frac{2 pi i log z}{log theta} right)
$$

where $log z = log |z| + i operatorname{Arg} z$ is the main branch of the logarithm with $0 < operatorname{Arg} z < pi$ for $z in Bbb H$.



All $log theta$ - periodic functions in $S$ are obtained by composing $g$ with a holomorphic function, this leads to the general solution
$$
f(z) = h left(exp left(frac{2 pi i log z}{log theta} right) right)
$$

where $h$ is holomorphic in the annulus ${ w mid exp left(frac{-2 pi^2}{log theta}right) < |w| < 1 }$.






share|cite|improve this answer











$endgroup$













  • $begingroup$
    Thank you so much! It's a great help!
    $endgroup$
    – chbe
    Jan 10 at 10:05
















1












$begingroup$

No, $f$ does not need to be constant.



Define $g$ in the strip $S = { w mid 0 < operatorname{Im} w < i pi } $ as
$g(w) = f(e^w)$. Then for a given positive real number $theta ne 1$
$$
f(z) = f(theta z) quad text{for all } z in Bbb H
$$

if and only if
$$
g(w) = g(w + log theta) quad text{for all } w in S
$$

so that the problem reduces to find all $log theta$ - periodic functions in the strip $S$. The most simple example would be
$$
g(w) = exp left(frac{2 pi i w}{log theta} right)
$$

corresponding to
$$
f(z) = exp left(frac{2 pi i log z}{log theta} right)
$$

where $log z = log |z| + i operatorname{Arg} z$ is the main branch of the logarithm with $0 < operatorname{Arg} z < pi$ for $z in Bbb H$.



All $log theta$ - periodic functions in $S$ are obtained by composing $g$ with a holomorphic function, this leads to the general solution
$$
f(z) = h left(exp left(frac{2 pi i log z}{log theta} right) right)
$$

where $h$ is holomorphic in the annulus ${ w mid exp left(frac{-2 pi^2}{log theta}right) < |w| < 1 }$.






share|cite|improve this answer











$endgroup$













  • $begingroup$
    Thank you so much! It's a great help!
    $endgroup$
    – chbe
    Jan 10 at 10:05














1












1








1





$begingroup$

No, $f$ does not need to be constant.



Define $g$ in the strip $S = { w mid 0 < operatorname{Im} w < i pi } $ as
$g(w) = f(e^w)$. Then for a given positive real number $theta ne 1$
$$
f(z) = f(theta z) quad text{for all } z in Bbb H
$$

if and only if
$$
g(w) = g(w + log theta) quad text{for all } w in S
$$

so that the problem reduces to find all $log theta$ - periodic functions in the strip $S$. The most simple example would be
$$
g(w) = exp left(frac{2 pi i w}{log theta} right)
$$

corresponding to
$$
f(z) = exp left(frac{2 pi i log z}{log theta} right)
$$

where $log z = log |z| + i operatorname{Arg} z$ is the main branch of the logarithm with $0 < operatorname{Arg} z < pi$ for $z in Bbb H$.



All $log theta$ - periodic functions in $S$ are obtained by composing $g$ with a holomorphic function, this leads to the general solution
$$
f(z) = h left(exp left(frac{2 pi i log z}{log theta} right) right)
$$

where $h$ is holomorphic in the annulus ${ w mid exp left(frac{-2 pi^2}{log theta}right) < |w| < 1 }$.






share|cite|improve this answer











$endgroup$



No, $f$ does not need to be constant.



Define $g$ in the strip $S = { w mid 0 < operatorname{Im} w < i pi } $ as
$g(w) = f(e^w)$. Then for a given positive real number $theta ne 1$
$$
f(z) = f(theta z) quad text{for all } z in Bbb H
$$

if and only if
$$
g(w) = g(w + log theta) quad text{for all } w in S
$$

so that the problem reduces to find all $log theta$ - periodic functions in the strip $S$. The most simple example would be
$$
g(w) = exp left(frac{2 pi i w}{log theta} right)
$$

corresponding to
$$
f(z) = exp left(frac{2 pi i log z}{log theta} right)
$$

where $log z = log |z| + i operatorname{Arg} z$ is the main branch of the logarithm with $0 < operatorname{Arg} z < pi$ for $z in Bbb H$.



All $log theta$ - periodic functions in $S$ are obtained by composing $g$ with a holomorphic function, this leads to the general solution
$$
f(z) = h left(exp left(frac{2 pi i log z}{log theta} right) right)
$$

where $h$ is holomorphic in the annulus ${ w mid exp left(frac{-2 pi^2}{log theta}right) < |w| < 1 }$.







share|cite|improve this answer














share|cite|improve this answer



share|cite|improve this answer








edited Jan 8 at 9:17

























answered Jan 8 at 8:59









Martin RMartin R

31k33561




31k33561












  • $begingroup$
    Thank you so much! It's a great help!
    $endgroup$
    – chbe
    Jan 10 at 10:05


















  • $begingroup$
    Thank you so much! It's a great help!
    $endgroup$
    – chbe
    Jan 10 at 10:05
















$begingroup$
Thank you so much! It's a great help!
$endgroup$
– chbe
Jan 10 at 10:05




$begingroup$
Thank you so much! It's a great help!
$endgroup$
– chbe
Jan 10 at 10:05











0












$begingroup$

If we write



$f(z) = f(x + iy) = u(x, y) + iv(x, y) = u(r, phi) + iv(r, phi), tag 1$



where $(r, phi)$ are standard polar coordinates on $Bbb H$, then we have



$dfrac{partial u}{partial r} + i dfrac{partial v}{partial r} = dfrac{partial f(z)}{partial r} = displaystyle lim_{Delta theta to 0} dfrac{f((1 + Delta theta)z) - f(z)}{Delta theta} = lim_{Delta theta to 0} dfrac{f(z) - f(z)}{Delta theta} = 0, tag 2$



since varying $theta$ moves $theta z$ in the radial direction; thus,



$dfrac{partial u}{partial r} = dfrac{partial v}{partial r} = 0; tag 3$



now according to the Cauchy-Riemann equations in the polar coordinates $(r, phi)$,



$ dfrac{partial v}{partial phi} = rdfrac{partial u}{partial r} = 0, tag 4$



$ dfrac{partial u}{partial phi} = -r dfrac{partial v}{partial r} = 0; tag 5$



these two equations together with (3) show that



$nabla u(z) = nabla v(z) = 0, forall z in Bbb H; tag 6$



it follows that $u(z)$ and $v(z)$ are constant in $Bbb H$, since it is a connected open set; thus so also $f(z)$ is constant on $Bbb H$.



The above derivation may be written in a slightly more compact fashion if we recall the polar form of the Cauchy-Riemann equations may be expressed in terms of the single equation in $f$,



$dfrac{partial f}{partial r} = dfrac{1}{ir} dfrac{partial f}{partial phi}; tag 7$



then (2) shows that



$dfrac{partial f}{partial r} = 0, tag 8$



from which we find that



$dfrac{partial f}{partial phi} = 0, tag 9$



and the constancy of $f(z)$ immediately follows.






share|cite|improve this answer









$endgroup$









  • 2




    $begingroup$
    I think that OP means that $f(theta z) = f(z)$ for all $z$ and some fixed $theta in Bbb R^+ - {1}$, not for all $z$ and all $theta$.
    $endgroup$
    – Travis
    Jan 8 at 9:06
















0












$begingroup$

If we write



$f(z) = f(x + iy) = u(x, y) + iv(x, y) = u(r, phi) + iv(r, phi), tag 1$



where $(r, phi)$ are standard polar coordinates on $Bbb H$, then we have



$dfrac{partial u}{partial r} + i dfrac{partial v}{partial r} = dfrac{partial f(z)}{partial r} = displaystyle lim_{Delta theta to 0} dfrac{f((1 + Delta theta)z) - f(z)}{Delta theta} = lim_{Delta theta to 0} dfrac{f(z) - f(z)}{Delta theta} = 0, tag 2$



since varying $theta$ moves $theta z$ in the radial direction; thus,



$dfrac{partial u}{partial r} = dfrac{partial v}{partial r} = 0; tag 3$



now according to the Cauchy-Riemann equations in the polar coordinates $(r, phi)$,



$ dfrac{partial v}{partial phi} = rdfrac{partial u}{partial r} = 0, tag 4$



$ dfrac{partial u}{partial phi} = -r dfrac{partial v}{partial r} = 0; tag 5$



these two equations together with (3) show that



$nabla u(z) = nabla v(z) = 0, forall z in Bbb H; tag 6$



it follows that $u(z)$ and $v(z)$ are constant in $Bbb H$, since it is a connected open set; thus so also $f(z)$ is constant on $Bbb H$.



The above derivation may be written in a slightly more compact fashion if we recall the polar form of the Cauchy-Riemann equations may be expressed in terms of the single equation in $f$,



$dfrac{partial f}{partial r} = dfrac{1}{ir} dfrac{partial f}{partial phi}; tag 7$



then (2) shows that



$dfrac{partial f}{partial r} = 0, tag 8$



from which we find that



$dfrac{partial f}{partial phi} = 0, tag 9$



and the constancy of $f(z)$ immediately follows.






share|cite|improve this answer









$endgroup$









  • 2




    $begingroup$
    I think that OP means that $f(theta z) = f(z)$ for all $z$ and some fixed $theta in Bbb R^+ - {1}$, not for all $z$ and all $theta$.
    $endgroup$
    – Travis
    Jan 8 at 9:06














0












0








0





$begingroup$

If we write



$f(z) = f(x + iy) = u(x, y) + iv(x, y) = u(r, phi) + iv(r, phi), tag 1$



where $(r, phi)$ are standard polar coordinates on $Bbb H$, then we have



$dfrac{partial u}{partial r} + i dfrac{partial v}{partial r} = dfrac{partial f(z)}{partial r} = displaystyle lim_{Delta theta to 0} dfrac{f((1 + Delta theta)z) - f(z)}{Delta theta} = lim_{Delta theta to 0} dfrac{f(z) - f(z)}{Delta theta} = 0, tag 2$



since varying $theta$ moves $theta z$ in the radial direction; thus,



$dfrac{partial u}{partial r} = dfrac{partial v}{partial r} = 0; tag 3$



now according to the Cauchy-Riemann equations in the polar coordinates $(r, phi)$,



$ dfrac{partial v}{partial phi} = rdfrac{partial u}{partial r} = 0, tag 4$



$ dfrac{partial u}{partial phi} = -r dfrac{partial v}{partial r} = 0; tag 5$



these two equations together with (3) show that



$nabla u(z) = nabla v(z) = 0, forall z in Bbb H; tag 6$



it follows that $u(z)$ and $v(z)$ are constant in $Bbb H$, since it is a connected open set; thus so also $f(z)$ is constant on $Bbb H$.



The above derivation may be written in a slightly more compact fashion if we recall the polar form of the Cauchy-Riemann equations may be expressed in terms of the single equation in $f$,



$dfrac{partial f}{partial r} = dfrac{1}{ir} dfrac{partial f}{partial phi}; tag 7$



then (2) shows that



$dfrac{partial f}{partial r} = 0, tag 8$



from which we find that



$dfrac{partial f}{partial phi} = 0, tag 9$



and the constancy of $f(z)$ immediately follows.






share|cite|improve this answer









$endgroup$



If we write



$f(z) = f(x + iy) = u(x, y) + iv(x, y) = u(r, phi) + iv(r, phi), tag 1$



where $(r, phi)$ are standard polar coordinates on $Bbb H$, then we have



$dfrac{partial u}{partial r} + i dfrac{partial v}{partial r} = dfrac{partial f(z)}{partial r} = displaystyle lim_{Delta theta to 0} dfrac{f((1 + Delta theta)z) - f(z)}{Delta theta} = lim_{Delta theta to 0} dfrac{f(z) - f(z)}{Delta theta} = 0, tag 2$



since varying $theta$ moves $theta z$ in the radial direction; thus,



$dfrac{partial u}{partial r} = dfrac{partial v}{partial r} = 0; tag 3$



now according to the Cauchy-Riemann equations in the polar coordinates $(r, phi)$,



$ dfrac{partial v}{partial phi} = rdfrac{partial u}{partial r} = 0, tag 4$



$ dfrac{partial u}{partial phi} = -r dfrac{partial v}{partial r} = 0; tag 5$



these two equations together with (3) show that



$nabla u(z) = nabla v(z) = 0, forall z in Bbb H; tag 6$



it follows that $u(z)$ and $v(z)$ are constant in $Bbb H$, since it is a connected open set; thus so also $f(z)$ is constant on $Bbb H$.



The above derivation may be written in a slightly more compact fashion if we recall the polar form of the Cauchy-Riemann equations may be expressed in terms of the single equation in $f$,



$dfrac{partial f}{partial r} = dfrac{1}{ir} dfrac{partial f}{partial phi}; tag 7$



then (2) shows that



$dfrac{partial f}{partial r} = 0, tag 8$



from which we find that



$dfrac{partial f}{partial phi} = 0, tag 9$



and the constancy of $f(z)$ immediately follows.







share|cite|improve this answer












share|cite|improve this answer



share|cite|improve this answer










answered Jan 8 at 0:37









Robert LewisRobert Lewis

48.9k23168




48.9k23168








  • 2




    $begingroup$
    I think that OP means that $f(theta z) = f(z)$ for all $z$ and some fixed $theta in Bbb R^+ - {1}$, not for all $z$ and all $theta$.
    $endgroup$
    – Travis
    Jan 8 at 9:06














  • 2




    $begingroup$
    I think that OP means that $f(theta z) = f(z)$ for all $z$ and some fixed $theta in Bbb R^+ - {1}$, not for all $z$ and all $theta$.
    $endgroup$
    – Travis
    Jan 8 at 9:06








2




2




$begingroup$
I think that OP means that $f(theta z) = f(z)$ for all $z$ and some fixed $theta in Bbb R^+ - {1}$, not for all $z$ and all $theta$.
$endgroup$
– Travis
Jan 8 at 9:06




$begingroup$
I think that OP means that $f(theta z) = f(z)$ for all $z$ and some fixed $theta in Bbb R^+ - {1}$, not for all $z$ and all $theta$.
$endgroup$
– Travis
Jan 8 at 9:06


















draft saved

draft discarded




















































Thanks for contributing an answer to Mathematics Stack Exchange!


  • Please be sure to answer the question. Provide details and share your research!

But avoid



  • Asking for help, clarification, or responding to other answers.

  • Making statements based on opinion; back them up with references or personal experience.


Use MathJax to format equations. MathJax reference.


To learn more, see our tips on writing great answers.




draft saved


draft discarded














StackExchange.ready(
function () {
StackExchange.openid.initPostLogin('.new-post-login', 'https%3a%2f%2fmath.stackexchange.com%2fquestions%2f3065559%2fholomorphic-function-on-a-upper-half-plane-that-is-scale-invariant-with-respect%23new-answer', 'question_page');
}
);

Post as a guest















Required, but never shown





















































Required, but never shown














Required, but never shown












Required, but never shown







Required, but never shown

































Required, but never shown














Required, but never shown












Required, but never shown







Required, but never shown







Popular posts from this blog

How do I know what Microsoft account the skydrive app is syncing to?

When does type information flow backwards in C++?

Grease: Live!